Skip to content

Commit ccff02e

Browse files
committed
update
1 parent 4c16a20 commit ccff02e

File tree

9 files changed

+292
-49
lines changed

9 files changed

+292
-49
lines changed

AlgebraicGeometry/1/solution/AlgebraicGeometry1.tex

Lines changed: 8 additions & 1 deletion
Original file line numberDiff line numberDiff line change
@@ -21,6 +21,7 @@
2121
\begin{problem}
2222
$P$ is an ideal of a unitary commutative ring $A$, then $P$ is prime ideal of $A\iff A/P$ is integral domain.
2323
\end{problem}
24+
\(\)
2425
\begin{solution}
2526
$\Rightarrow $:
2627

@@ -85,7 +86,7 @@
8586
\begin{solution}
8687
\begin{lemma}
8788
\label{lem:2}
88-
$K$ is a infinite field, $f\in K[x_1,x_2,\cdots x_n]\setminus \{0\}$, then exists $p\in \A_K^n,f(p)\neq 0$.
89+
$K$ is an infinite field, $f\in K[x_1,x_2,\cdots x_n]\setminus \{0\}$, then exists $p\in \A_K^n,f(p)\neq 0$.
8990
\end{lemma}
9091
\begin{proof}
9192
Use MI. When $n=0,K[x_1,x_2,\cdots x_n]=K$, so it's obvious. Assume for $n=k$ it's right, when goes to $k+1$:
@@ -102,5 +103,11 @@
102103
\(\phi\)
103104
So there is not a pair of points can be seperated by two disjoint open set.
104105
\end{solution}
106+
\begin{figure}
107+
\centering
108+
\includegraphics{~/Pictures/2-3.jpg}%![aaa](~/Pictures/2-3.jpg)
109+
110+
\end{figure}
105111

106112
\end{document}
113+

AlgebraicGeometry/10/solution/AlgebraicGeometry10.tex

Lines changed: 40 additions & 39 deletions
Original file line numberDiff line numberDiff line change
@@ -9,65 +9,66 @@
99
\large
1010
\setlength{\baselineskip}{1.2em}
1111
\ifpreface
12-
\input{../../../global/preface}
13-
\newgeometry{left=2cm,right=2cm,top=2cm,bottom=2cm}
12+
\input{../../../global/preface}
13+
\newgeometry{left=2cm,right=2cm,top=2cm,bottom=2cm}
1414
\else
15-
\newgeometry{left=2cm,right=2cm,top=2cm,bottom=2cm}
16-
\maketitle
15+
\newgeometry{left=2cm,right=2cm,top=2cm,bottom=2cm}
16+
\maketitle
1717
\fi
1818
%from_here_to_type
1919
\begin{problem}
20-
Assume \(V \subset \mathbbm{A}_k^{n}\) is irreducible, and \(p \in V\).
21-
Let \(m_p := \{f \in k[V] : f(p) = 0\}, M_p := \{f \in k[x_1,\cdots,x_n]: f(p) = 0\}\).
22-
Prove that \(m_p \cong M_p/\ide(V)\).
20+
Assume \(V \subset \mathbbm{A}_k^{n}\) is irreducible, and \(p \in V\).
21+
Let \(m_p := \{f \in k[V] : f(p) = 0\}, M_p := \{f \in k[x_1,\cdots,x_n]: f(p) = 0\}\).
22+
Prove that \(m_p \cong M_p/\ide(V)\).
2323
\end{problem}
24-
2524
\begin{solution}
2625
Consider \( \theta : M_p \to m_p ,\theta(f) := f + \ide(V)\).
27-
Since \( p \in V\) we get \(\theta\) is well-defined.
28-
And easily we get \(\theta\) is homomorphism. Now consider \(\ker \theta\).
29-
Obviously \(\ide(V) \subset \ker \theta\), now we prove \(\ker \theta \subset \ide(V)\).
30-
Assume \( f \in \ker\theta\), to prove \(f \in \ide(V)\).
31-
Since \(\theta(f)=\ide(V)\), we get \(f+\ide(V)=\ide(V)\), so \(f \in \ide(V)\).
32-
So \(\ker \theta = \ide(V)\). And easily \(\theta\) is surjective, so we get
33-
\(m_p=M_p/\ker \theta=M_p / \ide(V)\).
26+
Since \( p \in V\) we get \(\theta\) is well-defined.
27+
And easily we get \(\theta\) is homomorphism. Now consider \(\ker \theta\).
28+
Obviously \(\ide(V) \subset \ker \theta\), now we prove \(\ker \theta \subset \ide(V)\).
29+
Assume \( f \in \ker\theta\), to prove \(f \in \ide(V)\).
30+
Since \(\theta(f)=\ide(V)\), we get \(f+\ide(V)=\ide(V)\), so \(f \in \ide(V)\).
31+
So \(\ker \theta = \ide(V)\). And easily \(\theta\) is surjective, so we get
32+
\(m_p=M_p/\ker \theta=M_p / \ide(V)\).
3433
\end{solution}
3534
\begin{problem}
36-
Prove that \(M_p/M_p^2+\ide(V) \cong m_p/m_p^2\).
35+
Prove that \(M_p/M_p^2+\ide(V) \cong m_p/m_p^2\).
3736
\end{problem}
38-
3937
\begin{solution}
40-
Consider \(\theta : M_p \to m_p/m_p^2, f \mapsto f|_V+m_p^2\). Easily \(\theta\) is homomorphism and surjective.
41-
Now we prove \(\ker \theta = M_p^2+\ide(V)\).
38+
Consider \(\theta : M_p \to m_p/m_p^2, f \mapsto f|_V+m_p^2\). Easily \(\theta\) is homomorphism and surjective.
39+
Now we prove \(\ker \theta = M_p^2+\ide(V)\).
4240

4341
On one hand, assume \(f \in \ker \theta\), i.e., \(f|_V \in m_p^2\).
44-
Then \(\exists g_1,\cdots,g_n,h_1,\cdots,h_n \in m_p\) such that \(f|_V=\sum_{k=1}^{n} g_k h_k\).
45-
Assume \(g_k=G_k|_V,h_k=H_k|_V\). Then \(G_k,H_k \in M_p\).
46-
Consider \(f-\sum_{k=1}^{n} G_k H_k=:h \in k[x_1,\cdots,x_n]\), easily to know \(h(x)=0,\forall x \in V\).
47-
So \(h \in \ide(V)\), thus \(f \in M_p^2+\ide(V)\).
42+
Then \(\exists g_1,\cdots,g_n,h_1,\cdots,h_n \in m_p\) such that \(f|_V=\sum_{k=1}^{n} g_k h_k\).
43+
Assume \(g_k=G_k|_V,h_k=H_k|_V\). Then \(G_k,H_k \in M_p\).
44+
Consider \(f-\sum_{k=1}^{n} G_k H_k=:h \in k[x_1,\cdots,x_n]\), easily to know \(h(x)=0,\forall x \in V\).
45+
So \(h \in \ide(V)\), thus \(f \in M_p^2+\ide(V)\).
4846

49-
On the other hand, assume \(f \in M_p^2+\ide(V)\), to prove \(\theta f =0\).
50-
Assume \(f=\sum_{k=1}^{n} G_k H_k+h\), where \(G_k,H_k \in M_p\) and \(h \in \ide(V)\).
51-
Then \(\theta f = \sum_{k=1}^{n} g_k h_k +m_p^2\), where \(g_k=G_k|_V,h_k=H_k|_V\).
52-
So we get \(\theta f = m_p^2=0\).
47+
On the other hand, assume \(f \in M_p^2+\ide(V)\), to prove \(\theta f =0\).
48+
Assume \(f=\sum_{k=1}^{n} G_k H_k+h\), where \(G_k,H_k \in M_p\) and \(h \in \ide(V)\).
49+
Then \(\theta f = \sum_{k=1}^{n} g_k h_k +m_p^2\), where \(g_k=G_k|_V,h_k=H_k|_V\).
50+
So we get \(\theta f = m_p^2=0\).
5351

54-
Finally we get \(m_p/m_p^2 = M_p/ \ker \theta = M_p / M_p^2+\ide(V)\).
52+
Finally we get \(m_p/m_p^2 = M_p/ \ker \theta = M_p / M_p^2+\ide(V)\).
5553
\end{solution}
5654
\begin{problem}
57-
Assume \(V \subset \mathbbm{A}_k^n\) is irreducible, and \(p \in V,f \in k[V], f(p)\neq 0\).
58-
Consider \(V_f:= \{x \in V: f(x)\neq 0 \}\). Let \(\theta: V_f \to \mathbbm{A}_k^{n+1},x \mapsto (x,\frac{1}{f(x)})\).
59-
Let \(U = \theta(V_f)\), prove that \(T_p V \cong T_{\theta(p)} U\).
55+
Assume \(V \subset \mathbbm{A}_k^n\) is irreducible, and \(p \in V,f \in k[V], f(p)\neq 0\).
56+
Consider \(V_f:= \{x \in V: f(x)\neq 0 \}\). Let \(\theta: V_f \to \mathbbm{A}_k^{n+1},x \mapsto (x,\frac{1}{f(x)})\).
57+
Let \(U = \theta(V_f)\), prove that \(T_p V \cong T_{\theta(p)} U\).
6058
\end{problem}
6159

6260
\begin{solution}
63-
Write \(k[\mathbbm{A}_k^n]=k[x_1,\cdots , x_n ,y]\). Assume \(V = \van(I)= \van(f_1,\cdots, f_m)\), where \(f_i \in k[x_1,\cdots,x_n ]\).
64-
Then \(U = \van(f_1,\cdots,f_n, yf-1)\). Now consider \(\tau: T_p(V) \to \mathbbm{A}_k^{n+1}, \tau(x):=(x,\frac{1}{f(p)}-\frac{f_p^{(1)}(x)}{f^2(p)})\).
65-
Now we prove \(\tau(T_p V)=T_{\theta(p)} U\).
66-
Only need to prove \((yf-1)_{\theta(p)}^{(1)}(\tau(x))=0\).
67-
i.e., \(\frac{f_p^{(1)}(x)}{f(p)}+f(p)(y-\frac{1}{f(p)})=0\), where \(y = \frac{1}{f(p)}-\frac{f_p^{(1)}}{f^2(p)}\).
68-
Substitute \(y\) into the equation, we get is't obvious.
61+
Write \(k[\mathbbm{A}_k^n]=k[x_1,\cdots , x_n ,y]\). Assume \(V = \van(I)= \van(f_1,\cdots, f_m)\), where \(f_i \in k[x_1,\cdots,x_n ]\).
62+
Then \(U = \van(f_1,\cdots,f_n, yf-1)\). Now consider \(\tau: T_p(V) \to \mathbbm{A}_k^{n+1}, \tau(x):=(x,\frac{1}{f(p)}-\frac{f_p^{(1)}(x)}{f^2(p)})\).
63+
Now we prove \(\tau(T_p V)=T_{\theta(p)} U\).
64+
Only need to prove \((yf-1)_{\theta(p)}^{(1)}(\tau(x))=0\).
65+
i.e., \(\frac{f_p^{(1)}(x)}{f(p)}+f(p)(y-\frac{1}{f(p)})=0\), where \(y = \frac{1}{f(p)}-\frac{f_p^{(1)}}{f^2(p)}\).
66+
Substitute \(y\) into the equation, we get is't obvious.
6967

70-
Obviously \(\tau\) is injective, so it's isomorphic.
68+
Obviously \(\tau\) is injective, so it's isomorphic.
7169
\end{solution}
72-
70+
\begin{figure}
71+
\centering
72+
\includegraphics{~/Pictures/pic2/365/IMG_3135.JPG}
73+
\end{figure}
7374
\end{document}
5.58 KB
Binary file not shown.

MarkovProcess/14/solution/MarkovProcess14.tex

Lines changed: 24 additions & 1 deletion
Original file line numberDiff line numberDiff line change
@@ -28,7 +28,7 @@
2828
Easily \(N(t)+1 \geq T \geq N(t)\).
2929
So \(\int_{0}^{T}R(t) \d t \leq \sum_{i=1}^{N(T)+1} \int_{S_{i-1}}^{S_i}(S_i-t) \d t=\frac{1}{2}\sum_{i=1}^{N(t)+1} (S_i-S_{i-1})^2=\frac{1}{2} \sum_{i=1}^{N(T)+1} \xi_i^2\).
3030
For the same reason, we get that \(\int_{0}^{T}R(t) \d t \geq \frac{1}{2} \sum_{i=1}^{N(T)} \xi_i^2\).
31-
31+
\(\int_{0}^T R(t)\d t \geq \frac{1}{2}\sum_{i=1}^{N} \xi_i^2\)
3232
Easy to know that \(\lim_{T \to \infty} \frac{1}{T} \sum_{i=1}^{N(T)} \xi_i^2=\lim_{T \to \infty} \frac{1}{T} \sum_{i=1}^{N(T)+1} \xi_i^2=\frac{\mathbb{E}(X^2)}{\mathbb{E}(X)^2}\).
3333
So finally we get that
3434
\[
@@ -76,4 +76,27 @@
7676
So \(T=9.76\) can make the fee get minimum.
7777
\end{enumerate}
7878
\end{solution}
79+
\begin{problem}\label{pro:4 }
80+
A kind of product is qualified with probability \(p(0 < p < 1)\).
81+
We sample these product by the following way: we check all the product at first until there appears
82+
\(k\) qualified product sequently. Then we check the rest of product by probability \(\alpha(0 < \alpha <1)\)
83+
until there appears one unqualified product, then one circle ends. Next we restart another checking
84+
circle. Please find out the proportion of checked product after a long time.
85+
\end{problem}
86+
\begin{solution}
87+
For sake of convenience, we call the \(k\) qualified products appearing sequently as \(k\) qualified sequence.
88+
Assume the proportion of checked product after a long time is \(\beta\).
89+
Let \(N_k\) be the amount of product when the first \(k\) qualified sequence ends. \(M_k=\mathbb{E}(N_k)\). \(G_k\) is the
90+
event that the next one is qualified after the first \(k-1\) qualified sequence ends. Obviously, \(\mathbb{E}(N_k-N_{k-1} \mid G_k)=1\).
91+
And \(\mathbb{E}(N_k-N_{k-1} \mid G_k)=\mathbb{E}(N_k) + 1\).
92+
Therefore, \(\mathbb{E}(N_k-N_{k-1})=p + (1-p)(\mathbb{E}(N_k) + 1)\). So \(M_{k-} M_{k-1}=p + (1-p)( 1 + M_k)\).
93+
Then \(pM_k=M_{k-1} + 1\). Thus, \(M_k=\frac{\frac{1}{p^k}-1}{1-p}\).
94+
Let \(A=\{\)The amount of product checked in one circle\(\}\), \(B=\{\)The amount of product in one circle\(\}\).
95+
So finding one unqualified product need average checking time \(\frac{1}{1-p}\) according to geometric distribution.
96+
Then we averagely need \(\frac{1}{\alpha(1-p)}\) product to find out the unqualified one.
97+
Then \(\mathbb{E}(A)=M_k + \frac{1}{1-p}, \mathbb{E}(B)=M_k + \frac{1}{\alpha(1-p)}\).
98+
So \[
99+
\beta = \frac{\mathbb{E}(A)}{\mathbb{E}(B)}=\frac{\frac{\frac{1}{p^k}-1}{1-p} + \frac{1}{1-p}}{\frac{\frac{1}{p^k}-1}{1-p} + \frac{1}{\alpha(1-p)}}=\frac{\alpha}{\alpha + p^k-\alpha p^k}
100+
\]
101+
\end{solution}
79102
\end{document}

NumberTheory/8/solution/legendre.lua

Lines changed: 73 additions & 8 deletions
Original file line numberDiff line numberDiff line change
@@ -1,3 +1,11 @@
1+
local function isprime(n)
2+
for i = 2, n - 1 do
3+
if n % i == 0 then
4+
return false
5+
end
6+
end
7+
return true
8+
end
19
local function legendre(p, q)
210
if p % q == 0 then
311
return 0
@@ -16,9 +24,12 @@ local function legendre(p, q)
1624
return -1
1725
end
1826
local function defactor(n)
19-
local result = {}
27+
local result = { isprime = true }
2028
for i = 2, n, 1 do
2129
if n % i == 0 then
30+
if i < n then
31+
result.isprime = false
32+
end
2233
result[i] = 0
2334
while n % i == 0 do
2435
result[i] = result[i] + 1
@@ -42,17 +53,18 @@ local function termnext(term)
4253
return { { type = "number", value = (term.lower * term.lower - 1) % 16 == 0 and 1 or -1 } }
4354
end
4455
local factors = defactor(term.upper)
45-
if #factors == 1 then
56+
if factors.isprime then
4657
state = {
4758
{ type = "number", value = (term.upper % 4 == 1 or term.lower % 4 == 1) and 1 or -1 },
4859
{ type = "legendre", upper = term.lower, lower = term.upper },
4960
}
5061
return state
5162
end
5263
for k, v in pairs(factors) do
53-
print(k .. "," .. v)
54-
if v % 2 == 1 then
55-
state[#state + 1] = { type = legendre, upper = k, lower = term.lower }
64+
if type(v) == "number" then
65+
if v % 2 == 1 then
66+
state[#state + 1] = { type = "legendre", upper = k, lower = term.lower }
67+
end
5668
end
5769
end
5870
return state
@@ -67,16 +79,69 @@ local function term2str(term)
6779
str = [[\legendre{]] .. term.upper .. [[}{]] .. term.lower .. [[}]]
6880
end
6981
if term.type == "number" then
70-
print(term.value)
82+
-- print(term.value)
7183
str = tostring(term.value)
7284
end
7385
return str
7486
end
7587
local function state2str(state)
7688
local str = ""
89+
local number = 1
7790
for k, v in ipairs(state) do
78-
str = str .. term2str(v)
91+
if v.type == "legendre" then
92+
str = str .. term2str(v)
93+
end
94+
if v.type == "number" then
95+
number = number * v.value
96+
end
97+
end
98+
if str == "" then
99+
str = tostring(number)
100+
elseif number == -1 then
101+
str = "-" .. str
79102
end
80103
return str
81104
end
82-
print(state2str(termnext({ type = "legendre", upper = 6, lower = 13 })))
105+
local function statenext(oldstate)
106+
local newstate = { { type = "number", value = 1 } }
107+
local number = 1
108+
for _, v in ipairs(oldstate) do
109+
local tempstate = termnext(v)
110+
for _, y in ipairs(tempstate) do
111+
if y.type == "legendre" then
112+
newstate[#newstate + 1] = y
113+
elseif y.type == "number" then
114+
number = number * y.value
115+
end
116+
end
117+
end
118+
newstate[1]["value"] = number
119+
return newstate
120+
end
121+
local function legendre_with_calculation(p, q)
122+
if p % q == 0 then
123+
return 0
124+
end
125+
if q == 0 then
126+
error("the second argument of legendre should be non-zero!")
127+
end
128+
if q < 0 then
129+
q = -q
130+
end
131+
local state = { { type = "legendre", upper = p, lower = q } }
132+
local calculation = "\\[\n" .. state2str(state)
133+
while #state > 1 or state[1].type == "legendre" do
134+
state = statenext(state)
135+
calculation = calculation .. "\\\\\n=" .. state2str(state)
136+
end
137+
calculation = calculation .. "\n\\]"
138+
return calculation
139+
end
140+
for i = 1, 100, 1 do
141+
local p = math.random(1, 1000)
142+
local q = math.random(1001, 10000)
143+
while not isprime(q) do
144+
q = math.random(1001, 10000)
145+
end
146+
print(legendre_with_calculation(p, q))
147+
end

WaterClass/3/.number

Lines changed: 1 addition & 0 deletions
Original file line numberDiff line numberDiff line change
@@ -0,0 +1 @@
1+
3

WaterClass/3/solution/WaterClass3.pdf

103 KB
Binary file not shown.

0 commit comments

Comments
 (0)